Cardiac Rhythm Devices : A Case-Based Approach to Management [1 ed.] 9781935281399, 9781933864679

The rapid pace of device development and complexity in applications along with a lack of concise, problem-oriented refer

147 29 6MB

English Pages 194 Year 2011

Report DMCA / Copyright

DOWNLOAD PDF FILE

Recommend Papers

Cardiac Rhythm Devices : A Case-Based Approach to Management [1 ed.]
 9781935281399, 9781933864679

  • 0 0 0
  • Like this paper and download? You can publish your own PDF file online for free in a few minutes! Sign Up
File loading please wait...
Citation preview

R H Y T HM DE V ICE S A Case-Based Approach to Management Mehdi Razavi, MD, Alireza Nazeri, MD, Ali Massumi, MD, and Samuel J. Asirvatham, MD The rapid pace of device development and complexity in applications along with a lack of concise, problem-oriented references has made it difficult for non-electrophysiologists to confidently manage patients with implanted cardiac rhythm devices (ICRDs).

Nearly fifty in-depth case presentations provide real-world examples of issues associated with IRCDs and the many factors involved in clinical decision-making and effective management. The cases can be studied individually and in any order. Also included are handy indexes of “teaching points” that make it easy to quickly locate cases and discussions of any specific topic. This practical book is an indispensable resource and quick reference for the clinical cardiologist, cardiology fellow, or any practitioner needing to improve understanding and skills in the application of IRCDs. ABOUT THE AUTHORS Mehdi Razavi, MD, Director Clinical Arrhythmia Research, Texas Heart Institute and Assistant Clinical Professor of Medicine, Baylor College of Medicine, Houston, Texas Alireza Nazeri, MD, Senior Cardiology Research Fellow, Texas Heart Institute, Houston, Texas Ali Massumi, MD, Sultan Qabus Chair in Cardiac Electrophysiology, and Director, Center for Cardiac Arrhythmias and Electrophysiology, Texas Heart Institute, Houston, Texas Samuel J. Asirvatham, MD, Consultant, Division of Cardiovascular Diseases and Internal Medicine, Division of Pediatric Cardiology and Professor of Medicine, Mayo Clinic College of Medicine, Rochester, Minnesota Recommended Shelving Category: Cardiology

11 West 42nd Street New York, NY 10036-8002 www.demosmedpub.com

9 781933 864679

CARDIAC RHYTHM DEVICES A Case-Based Approach to Management

Cardiac Rhythm Devices: A Case-Based Approach to Management provides a practical, problem-oriented, and case-based guide to assist in troubleshooting and management of the common complications of ICRDs. Readers will gain fundamental understanding of the applications and clinical indications, procedural techniques, pre- and postprocedure management, and common complications and their management.

Razavi • Nazeri • Massumi • Asirvatham

CA R DI A C

CA RDIAC

RH Y T HM DE V ICE S A Case-Based Approach to Management Foreword by David Hayes

Mehdi Razavi Alireza Nazeri Ali Massumi Samuel J. Asirvatham

Cardiac Rhythm Devices A Case-Based Approach to Management

Cardiac Rhythm Devices A Case-Based Approach to Management Mehdi Razavi, MD Director Clinical Arrhythmia Research Texas Heart Institute Assistant Clinical Professor of Medicine Baylor College of Medicine Houston, Texas Alireza Nazeri, MD Senior Cardiology Research Fellow Texas Heart Institute Houston, Texas Ali Massumi, MD Sultan Qabus Chair in Cardiac Electrophysiology Director Center for Cardiac Arrhythmias and Electrophysiology Texas Heart Institute Houston, Texas Samuel J. Asirvatham, MD Consultant Division of Cardiovascular Diseases and Internal Medicine Division of Pediatric Cardiology Professor of Medicine Mayo Clinic College of Medicine Rochester, Minnesota

New York

Acquisitions Editor: Richard Winters Cover Design: Steve Pisano Compositor: The Manila Typesetting Company Printer: Hamilton Printing Company Visit our website at www.demosmedpub.com © 2011 Demos Medical Publishing, LLC. All rights reserved. This book is protected by copyright. No part of it may be reproduced, stored in a retrieval system, or transmitted in any form or by any means, electronic, mechanical, photocopying, recording, or otherwise, without the prior written permission of the publisher. Medicine is an ever-changing science. Research and clinical experience are continually expanding our knowledge, in particular our understanding of proper treatment and drug therapy. The authors, editors, and publisher have made every effort to ensure that all information in this book is in accordance with the state of knowledge at the time of production of the book. Nevertheless, the authors, editors, and publisher are not responsible for errors or omissions or for any consequences from application of the information in this book and make no warranty, express or implied, with respect to the contents of the publication. Every reader should examine carefully the package inserts accompanying each drug and should carefully check whether the dosage schedules mentioned therein or the contraindications stated by the manufacturer differ from the statements made in this book. Such examination is particularly important with drugs that are either rarely used or have been newly released on the market. Library of Congress Cataloging-in-Publication Data Cardiac rhythm devices : a case-based approach to management / Mehdi Razavi ... [et al.]. p. ; cm. Includes bibliographical references and index. ISBN 978-1-933864-67-9 1. Cardiac pacing--Case studies. I. Razavi, Mehdi, M.D. [DNLM: 1. Pacemaker, Artificial--contraindications--Case Reports. 2. Arrhythmias, Cardiac--diagnosis--Case Reports. 3. Arrhythmias, Cardiac--therapy--Case Reports. 4. Cardiac Pacing, Artificial--Case Reports. WG 26 C2695 2011] RC684.P3C318 2011 617.4’120645--dc22 2010024855 Special discounts on bulk quantities of Demos Medical Publishing books are available to corporations, professional associations, pharmaceutical companies, health care organizations, and other qualifying groups. For details, please contact: Special Sales Department Demos Medical Publishing 11 W. 42nd Street, 15th Floor New York, NY 10036 Phone:  800–532–8663 or 212–683–0072 Fax:  212–941–7842 E-mail:  [email protected] Made in the United States of America 10  11  12  13  14   5  4  3  2  1

Contents Foreword Preface Teaching Points

ix xi xiii

Pacemakers (PM) PM Teaching Points

3

    PM 1   Recurrent Syncope after Pacemaker Implantation

5

    PM 2   Palpitations after Dual-chamber Pacemaker

9

    PM 3   A 72-year-old Man with a Pacemaker and Presyncope

13

    PM 4  A 52-year-old Man with Possible Pacemaker Malfunction and Syncope

17

    PM 5  Device Management after Atrioventricular Node Ablation in an 82-year-old Man

21

    PM 6  A 75-year-old Woman with a Pacemaker and Syncope during Prayer

23

    PM 7   A 5 4-year-old Man with Permanent Pacemaker Presents with Fever

27

    PM 8   A 56-year-old Man with Possible Device Malfunction

31

    PM 9   A 73-year-old Woman with Possible Pacemaker Noncapture

33

   PM 10   Multiple Pacing Artifacts in a Critically Ill Patient

35

   PM 11  A 74-year-old Man with Dual-chamber Pacemaker and Shortness of Breath

37

   PM 12  Ventricular Tachycardia in a Patient with a Dual-chamber Pacemaker

41

   PM 13   Ventricular Tachycardia Detected during Pacemaker Interrogation

45

   PM 14  Intermittent Atrial Pacing in a 62-year-old Man with a Dual-chamber Pacemaker

49

   PM 15   Tachycardia during Pacing

51

   PM 16   Onset of Ventricular Tachycardia during Pacemaker Threshold Testing

55 

Contents

   PM 17  Dual-chamber Pacemaker with Possible Lead Malfunction

59

   PM 18   Inflammatory Reaction on Device Site

61

Implantable Cardiac Defibrillators (ICD) ICD Teaching Points

67

   ICD 1    Exercise-induced Implantable Cardioverter Defibrillator Shocks

69

   ICD 2    Multiple Implantable Cardioverter Defibrillator Shocks in a 45-year-old Man

73

   ICD 3    Interrogation of a Dual-chamber Implantable Cardioverter Defibrillator

75

   ICD 4    Long-QT Syndrome and Multiple Implantable Cardioverter Defibrillator Shocks

79

   ICD 5    Frequent Palpitations in a Patient with Implantable Cardioverter Defibrillator

83

   ICD 6    Palpitations in a Patient with a Dual-chamber Implantable Cardioverter Defibrillator

87

   ICD 7    Variable Pacing in a Patient with Congenital Atrioventricular Block

91

   ICD 8    Multiple Implantable Cardioverter Defibrillator Shocks

93

   ICD 9    Rapid Analysis of Shock Etiology

95

  ICD 10    Wide Complex Tachycardia with Periodic Oscillation

97

  ICD 11    Ventricular Ectopy on a Rhythm Strip

99

  ICD 12    Ventricular Pacing and Atrial Flutter

101

  ICD 13    Ventricular Tachycardia Undersensing in a Dual-chamber Implantable Cardioverter Defibrillator 103   ICD 14   A 52-year-old Man with a Beeping Implantable Cardioverter Defibrillator

105

Biventricular Device Troubleshooting (BiV) BiV Teaching Points

111

     BiV   Case Series 1 Case 1.1   Locating the Left Ventricular Pacing Lead — Electrocardiographic Fluoroscopic Correlation vi

113

Contents  

Case 1.2   Will Offset Help?

116

Case 1.3   Not Allowing Cardiac Resynchronization

119

Case 1.4   Role of the Atrial Lead

122

     BiV   Case Series 2 Case 2.1   Why Does the Morphology Change?

127

Case 2.2   Capture with Delay

129

     BiV   Case Series 3 Case 3.1   Troubleshooting Difficulty with Left Ventricular Lead Implantation

133

Case 3.2   Difficulties Continued

136

     BiV   Case Series 4 Case 4.1  Troubleshooting Failure to Improve— Electrocardiographic Correlation

141

Case 4.2   Programming Atrioventricular Interval

143

     BiV   Case Series 5 Case 5.1   Cardiac Resynchronization—Handling Complexity

147

Case 5.2   Handling Complexity

148

Case 5.3   Still Not Responding

149

Case 5.4   Epicardial Pacing

151

Case 5.5   Varying QRS Width and Morphology

152

   BiV 6   Analysis of Tracings during Device Interrogation in a Patient with a Biventricular Implantable Cardioverter Defibrillator

155

   BiV 7   Fusion and Pseudofusion

157

   BiV 8   Multiple Leads on Chest Radiography

159

Further Reading Index

165 167

vii

Foreword The field of medicine has gone beyond the longstanding prescribed requirement of Continuing Medical Education as a method to maintain competence in a given field. We are now focused on ‘life-long learning’ in an effort to provide the best patient care in a discipline that is constantly changing and also for the purposes of credentialing and re-credentialing in our primary specialty, sub-specialties, and sub-sub-specialties. Many would agree that there is no better way to maintain one’s clinical skills and to prepare for credentialing and re-credentialing examinations than to work through ‘real-world’ cases. Dr. Razavi and colleagues provide such real-world case examples in the discipline of implantable cardiac rhythm devices in this handbook. They have provided a broad selection of clinical cases that cover pacemakers, implantable cardioverter defibrillators, and cardiac resynchronization therapy devices. The reader has the ability to approach the case as an unknown as if they were seeing the patient, review the pertinent tracings and/or telemetry, and decide on the best approach to management. This is followed by a discussion of the important clinical points that should be considered to optimize patient management. This handbook will be helpful to any caregiver involved in the management of implantable cardiac rhythm devices for the purpose of providing improved patient care as well as for preparation of credentialing and re-credentialing examinations. David Hayes, MD Division of Cardiovascular Diseases Professor of Medicine Mayo Clinic College of Medicine Mayo Clinic Rochester, Minnesota

ix

Preface The recent increase in utilization of implantable cardiac devices has made it ever more important for clinicians to have a fundamental understanding of this technology. While numerous comprehensive textbooks are available for an exhaustive study of the field, there remains a paucity of literature devoted to a detailed analysis of “real-life” intracardiac device tracings. We believe that the process of understanding the concepts represented in these tracings will enable the clinician to grasp the fundamentals underlying cardiac devices in an enjoyably challenging manner. This will enhance retention and comprehension of what is often and incorrectly thought of as an abstruse field. Each of the tracings in this collection can be studied individually and in any order. For each tracing we have focused on the main interpretation in the hopes of discussing the most relevant clinical problem a clinician may encounter in the field It is our hope that after reading each case the readers will give themselves an opportunity to formulate a response before moving on to the discussion for the specific case. References are provided for those readers interested in a more in-depth review of the field. This book is a quick reference for physicians and in-training house staff in the fields of cardiology as well as family medicine, internal medicine, and emergency medicine, and aims to assist in the diagnosis and management of common device-related clinical problems. Mehdi Razavi, MD Alireza Nazeri, MD Ali Massumi, MD Samuel J. Asirvatham, MD

xi

Teaching Points

Abbreviations: PM, Pacemaker; ICD, Implantable Cardiac Defibrillator; BiV, Biventricular.

Arrhythmia detection algorithms, ICD Case 2, ICD Case 8 Arrhythmia diagnosis algorithm, ICD Case 5 Arrhythmia sensing, BiV Case 6 Atrial fibrillation, PM Case 14 Atrial high-rate events, ICD Case 6 Atrial lead, BiV Case 1.4 Atrial tachyarrhythmias, ICD Case 12 Atrial undersensing, PM Case 16 Audible alerts, ICD Case 14 AV block, PM Case 14 AV node ablation, PM Case 5 Biventricular pacing, BiV Case 7 Blanking periods, ICD Case 1 Capture delay, BiV Case 2.2 Cardiac arrhythmias in patients with pacemakers, PM Case 15 Cardiac resynchronization: handling complexity, BiV Case 5.1, 5.2 Cardiac resynchronization: not responding, BiV Case 5.3 Crosstalk, PM Case 4, PM Case 8 Detection algorithms, ICD Case 11 Device diagnostics, ICD Case 9 Device infection, PM Case 7 Device malfunction, PM Case 2, PM Case 6, PM Case 11 Device troubleshooting, ICD Case 8, ICD Case 14, PM Case 5

Device-device interactions, PM Case 10 Double counting, ICD Case 1, ICD Case 8, ICD Case 9 Epicardial pacing, BiV Case 5.4 Failure to capture, BiV Case 7 Failure to improve: electrocardiographic correlation, BiV Case 4.1 Fusion, ICD Case 3, BiV Case 7 High-rate episodes, PM Case 13 Hypersensitivity to device composites, PM Case 18 Inappropriate ICD shocks, ICD Case 2, ICD Case 4, ICD Case 9 Latency, BiV Case 6 Lead fracture, ICD Case 4, PM Case 6 Lead localization on radiography, BiV Case 8 Locating LV pacing lead, BiV Case 1.1 Lead malfunction, ICD Case 9 Lower rate limit, PM Case 1 LV lead implantation difficulty, BiV Case 3.1, BiV Case 3.2 Managed ventricular pacing, PM Case 17 Management of device allergy, PM Case 18 Marker channel analysis, PM Case 13 Mode switching, ICD Case 6, ICD Case 10, ICD Case 12, PM Case 11, PM Case 14

xiii

Teaching Points

Morphology, ICD Case 11 Morphology change, BiV Case 2.1 Noise, ICD Case 4, PM Case 12 Noncapture, ICD Case 3, PM Case 9, PM Case 10 Not allowing cardiac resynchronization, BiV Case 1.3 Offset, BiV Case 1.2 Oversensing, ICD Case 1, PM Case 4, PM Case 6, PM Case 8, PM Case 9, PM Case 11, PM Case 12 Pacemaker dependent patients, PM Case 10 Pacemaker lead malfunction, PM Case 15 Pacemaker malfunction, PM Case 4, PM Case 12 Pacemaker mediated tachycardia, PM Case 2, ICD Case 13 Pacemaker-programmed intervals, PM Case 3, PM Case 4 Pacing inhibition, ICD Case 10, PM Case 8 Pacing timing intervals, ICD Case 6 PMT, PM Case 16 Post ventricular atrial refractory period, PM Case 2 Programming AV interval, BiV Case 4.2

xiv

Pseudofusion, BiV Case 7, ICD Case 3 Rate drop feature, PM Case 1 Sensing, ICD Case 8, ICD Case 11, ICD Case 12, PM Case 1 Sinus capture, ICD Case 10 Sudden death, PM Case 5 Syncope, PM Case 1 Threshold testing, BiV Case 6, PM Case 9 Total atrial refractory period (TARP), ICD Case 7 Tracking, ICD Case 12 Troubleshooting, PM Case 16 T-wave oversensing, ICD Case 1, ICD Case 8, ICD Case 9 Undersensing, PM Case 10, PM Case 14 Upper rate behavior, ICD Case 12, PM Case 3 Variable atrial pacing, ICD Case 7 Varying QRS width and morphology, BiV Case 5.5 Ventricular safety pacing, PM Case 8 Ventricular sensing and capture, BiV Case 6 Ventricular tachycardia, PM Case 13 “Wobble” phenomenon, ICD Case 5

Cardiac Rhythm Devices A Case-Based Approach to Management

Pacemakers

Pacemaker Teaching Points (PM)

Atrial fibrillation, PM Case 14 Atrial undersensing, PM Case 16 AV block, PM Case 14 AV node ablation, PM Case 5 Cardiac arrhythmias in patients with pacemakers, PM Case 15 Crosstalk, PM Case 4, PM Case 8 Device infection, PM Case 7 Device malfunction, PM Case 2, PM Case 6, PM Case 11 Device troubleshooting, PM Case 5 Device-device interactions, PM Case 10 High-rate episodes, PM Case 13 Hypersensitivity to device composites, PM Case 18 Lead fracture, PM Case 6 Lower rate limit, PM Case 1 Managed ventricular pacing, PM Case 17 Management of device allergy, PM Case 18 Marker channel analysis, PM Case 13 Mode switching, PM Case 14 Noise, PM Case 12 Noncapture, PM Case 9, PM Case 10 Oversensing, PM Case 4, PM Case 6, PM Case 8, PM Case 9, PM Case 11, PM Case 12

Pacemaker dependent patients, PM Case 10 Pacemaker lead malfunction, PM Case 15 Pacemaker malfunction, PM Case 4, PM Case 12 Pacemaker mediated tachycardia, PM Case 2 Pacemaker programmed intervals, PM Case 3, PM Case 4 Pacing inhibition, PM Case 8 PMT, PM Case 16 Post ventricular atrial refractory period, PM Case 2 Rate drop feature, PM Case 1 Sensing, PM Case 1 Sudden death, PM Case 5 Syncope, PM Case 1 Threshold testing, PM Case 9 Troubleshooting, PM Case 16 Undersensing, PM Case 10, PM Case 14 Upper rate behavior, PM Case 3 Ventricular safety pacing, PM Case 8 Ventricular tachycardia, PM Case 13



Case 1  R  ecurrent Syncope after Pacemaker Implantation TEACHING POINTS • Rate drop feature • Lower rate limit

• Sensing • Syncope

Case Presentation A 53-year-old man underwent implantation of a dual-chamber permanent pacemaker 10 days ago for recurrent palpitations. The episodes were occurring on the average of once every 2 to 3 weeks and were associated with a prodrome of flushing and nausea. All of these occurred while he was working as a cashier in a supermarket. The physical examination, electrocardiogram, echo and stress test results had been normal. A head upright tilt-table (HUT) test had been performed. The results demonstrated a sudden drop in heart rate 17 min into the study. A diagnosis of neuorcardiogenic syncope with a predominant cardioinhibitory response was made. He underwent implantation of a dualchamber pacemaker and was discharged without complication. He now presents with two episodes of syncope in the last two days. They are associated with the same prodromes as previous. The prodromes last longer, and this gives him the chance to lay supine before loss of consciousness. The syncope itself lasted less than 10 s according to witnesses. Recovery was spontaneous. The patient is asymptomatic at this time. Examination is unremarkable except for mild orthostasis (supine blood pressure, 102/64 mm Hg; recumbent blood pressure, 94/58 mm Hg). The supine rhythm and heart rate are sinus and 92 beats/min, respectively. Upon standing, however, it is noted that the patient’s rhythm



Case 1  |  Recurrent Syncope after Pacemaker Implantation

becomes paced as his heart rate drops to 60 beats/min. The rhythm is atrial paced with intrinsic ventricular conduction at rate of 120 beats/min. Interrogation of the pacemaker demonstrates the following settings: DDD, 50 to 130 beats/min AV (sensed), 180 msec AV delay (paced), 200 msec The tracing is shown below (Figure 1):

figure 1

Case Discussion Is the device working appropriately? Why was the rate response feature not turned on? Why did the patient have recurrent syncope? Neurocardiogenic syncope (NCS) is defined as syncope caused by dysregulation of the autonomic nervous system. It is caused by inappropriate response to orthostatic challenge. It is manifested by a continuum between two extremes: cardioinhibitory response describes the condition wherein the primary response is a drop in heart rate induced by orthostasis, or other physiological stressors. Vasodepressor response is defined by a state of relative or absolute vasodilation in which vascular resistance, particularly of the venous capacitance vessels in the lower extremities, becomes impaired.



Case 1  |  Recurrent Syncope after Pacemaker Implantation

In both cases, the result is impaired central perfusion with secondary symptoms of light-headedness and syncope. Most commonly, patients have a combination of these derangements, which may present with different extremes of each at different times. The underlying trigger, however, is usually orthostatic stress brought on by real or relative intravascular depletion. Thus, all patients with NCS should undergo conservative management in the form of aggressive hydration, use of lower extremity compression stockings, and sodium repletion if their blood pressure allows this. In the current case, the results of the HUT demonstrated a drop in heart rate followed by syncope. Presumably, this cardioinhibitory response would be curable with a pacemaker. As noted above, however, patients may present with a combination or variety of response types in NCS. The patient’s history, physical examination, and investigations were suggestive of NCS: a “vagal” prodrome of nausea and flushing while in the recumbent position without evidence of structural heart disease. The presenting physiological abnormality cannot be reliably predicted by a HUT. Thus, despite placement of a pacemaker, attention still should be given to maintaining intravascular volume using the measures noted above. It may have even been reasonable to observe the patient’s response to volume depletion before pacemaker implantation. Although studies have shown pacemakers to decrease the incidence of syncope, they cannot eliminate all such events. The tracing demonstrates sinus rhythm with onset of pacing at 850 msec. This is shorter than the lower rate limit (LRL) of the pacemaker (1000 msec). The reason for this is that the pacemaker has a rate drop mode programmed on. In this case, the device is programmed to detect a sudden decrease in heart rate. This decrease is defined by the device and is programmable. The most common definitions incorporate the change in cycle length over a specified period of time. In this example, the device was programmed to “detect” a drop in rate if there was a decrease of 20 beats/min over a maximum 30-s period. We can see onset of atrial pacing (star) at a higher rate once the “trigger” (vertical line) is declared. Thus, any decrease of 20 beats/min occurring over a period of less than 30 s triggered the feature. The response of the



Case 1  |  Recurrent Syncope after Pacemaker Implantation

feature is also programmable but usually incorporates pacing at a higher rate for a period of time. It should be noted that the heart rate need not drop below the lower rate limit of the device to declare a “rate drop” event. As such, pacing can occur at rate faster (or cycle length shorter) than the LRL, as seen in this case. The upper rate of pacing, however, does not violate the upper rate limit (URL). As such, the pacemaker demonstrated normal function.



Case 2  Palpitations after Dual-chamber Pacemaker TEACHING POINTS • Pacemaker-mediated tachycardia • Postventricular atrial refractory period

• Device malfunction

Case Presentation A 62-year-old man is referred for ventricular tachycardia. He states that for the past two weeks he has been having frequent palpitations. Some of these last for minutes. The onset and termination are abrupt. There is no associated chest discomfort, shortness of breath, or light-headedness. The episodes started two weeks after placement of a dual-chamber pacemaker. The pacemaker was placed because of symptomatic bradycardia. He has no history of coronary artery disease or cardiomyopathy. His only medication is aspirin. His examination is unremarkable. His resting electrocardiogram demonstrates sinus bradycardia. You are asked to evaluate his rhythm. Interrogation of his pacemaker shows no evidence of ventricular high rate episodes. The device settings are as follows: DDDR, 60 to 120 beats/min Interrogation of his pacemaker during one of the episodes of wide complex tachycardia shows the tracing in Figure 1 on the following page.



Case 2  |  Palpitations after Dual-chamber Pacemaker

figure 1

10

Case 2  |  Palpitations after Dual-chamber Pacemaker

Case Discussion Pacemaker-mediated tachycardia (PMT) is a form of endless loop tachycardia wherein an atrial sensed event leads to ventricular tracking (pacing). The ventricular impulse then travels in a retrograde direction, usually via the atrioventricular node, to the atrium. The atrial activation is then sensed by the atrial lead. This again leads to ventricular pacing, retrograde conduction of ventricular activation, and repetition of the circuit. The common element in initiation of PMT is retrograde conduction of a ventricular impulse. The impulse can be due to ventricular pacing or intrinsic ventricular activation. A second requirement is presence of intrinsic ventriculo-atrial (VA) conduction. The VA conduction allows the atrial ventricular impulse to travel to the atrium. Once the atrial activation is sensed, ventricular tracking via the pacemaker follows (because the AV node will likely be refractory from the previous VA conduction). The ventricle is then paced. By this time, the AV node has recovered, once again allowing retrograde conduction and persistence of the tachycardia. Thus, the most common clinical triggers for PMT are premature ventricular contractions, as they can lead to VA conduction, atrial sensing, ventricular tracking, and onset of tachycardia. Atrial undersensing can lead to PMT by leading to subsequent atrial pacing (without capture). Because the atrium does not capture, there is no AV conduction and ventricular tracking will ensue. The ventricular paced impulse can then conduct retrogradely to initiate PMT. Premature atrial contractions can be sensed, block in the AV node, and lead to ventricular pacing and initiation of PMT. To reiterate, any conduction associated with ventricular activation in the setting of an AV node that allows retrograde conduction is conducive for initiation of PMT. In the tracing, sensed atrial events are followed by V-paced events (Fig­ ure 2 corresponds to the middle panels on the tracing in Figure 1). The atrial sensed events fall outside of the postventricular atrial refractory period (PVARP). Atrial sensed events falling within the PVARP are recognized but are not used to initiate an AV interval. Thus, ventricular tracking does not occur. This protects against the initiation of PMT. Extending the PVARP will increase the likelihood of the atrial sensed event falling within the PVARP and thus not being tracked. 11

Case 2  |  Palpitations after Dual-chamber Pacemaker

figure 2

Placing a magnet on a pacemaker will eliminate all sensing. The pacemaker essentially starts pacing at the preset rate regardless of all external events. As noted, propagation of PMT depends on atrial sensing. Thus, when a magnet is placed on the pacemaker, the circuit is eliminated (atrial sensed event no longer leads to ventricular pacing). This terminates PMT and is indeed one of its diagnostic features. Atrial tachycardia with rapid tracking is not effected by a magnet.

12

Case 3  A  72-year-old Man with a Pacemaker and Presyncope TEACHING POINTS • Upper rate behavior

• Pacemaker programmed intervals

Case Presentation A 72-year-old man presents for routine pacemaker check. He received a dualchamber pacemaker for severe AV nodal conduction disease. The device parameters are as follows: DDDR’s LRL, 50 beats/min URL, 130 beats/min Sensed atrioventricular interval (AVI), 250 msec Paced AVI, 270 msec Postventricular atrial refractory period (PVARP), 330 msec He informs you of recent onset of severe light-headedness brought on by vigorous exercise. This occurs during peak exercise, usually when his heart rate is in the range of 120 to 140 beats/min based on his wrist meter. He suddenly feels flushed and faint at the same time the monitor shows a rate of 50 to 70 beats/min. He had not had any problems until recently when the b-blocker dose he was taking for the management of hypertension was decreased. You decide to have the patient exercise on the treadmill.

13

Case 3  |  A 72-year-old Man with a Pacemaker and Presyncope

figure 1

Case Discussion What response do you expect to see as the sinus node accelerates? At what rate would this response be noted? The upper rate behavior of a pacemaker is defined as the response to atrial sensed events in a patient whose AV conduction is dependent on the pacemaker. Two responses are notable: AV Wenckebach is a pattern in which the URL is achieved in the ventricle. In this scenario, the URL is reached in the ventricle because all atrial events are sensed (ie, are not in a refractory period). The relative frequency of nonconducted atrial events is a function of the ratio between these events and the URL of the pacemaker. Faster atrial events impose more frequent block of these events as the pacemaker cannot exceed the LRL. Wenckebach periodicity as defined by a shorter AV interval after the dropped beat than before the dropped beat is not observed, nor is progressive shortening of R-R intervals before a dropped beat. A 2:1 AV block can abruptly occur if at a certain atrial rate, every other beat occurs during a refractory period. In this case, at a certain atrial cycle length, every other beat falls in the total atrial refractory period (TARP), defined as the combination of the AV interval and PVARP. The clinical implication is important: once a patient’s atrial cycle length (CL) becomes short enough so that every other beat is in the TARP, those

14

Case 3  |  A 72-year-old Man with a Pacemaker and Presyncope

beats fail to be sensed and tracked. This, in effect, leads to 2:1 AV block. The sudden drop in heart rate from the maximum tracking rate to one half that rate can be very symptomatic. Shortening of the TARP requires short atrial CL (increased rates) for the onset of this phenomenon. This can be done by shortening either or both the AVI or PVARP. There can be a transition from AV Wenckebach upper rate behavior to 2:1 block. This transition occurs when the atrial CL becomes shorter than TARP. The rate at which this phenomenon occurs can be calculated by calculating the TARP. Thus, in this patient, TARP is 580 msec and 2:1 AV block would occur once the atrial cycle length shortens below this. In this patient, very likely, withholding b-blockade enabled a higher sinus rate to be achieved. The cycle length shortened below TARP and symptomatic 2:1 block ensued. The periodic nature of the ventricular paced rate may be explained by the fact that sensed P-waves close to the preceding paced QRS were in the post-ventricular refractory period.

15

Case 4  A  52-year-old Man with Possible Pacemaker Malfunction and Syncope TEACHING POINTS • Crosstalk • Oversensing

• Pacemaker malfunction • Pacemaker programmed intervals

Case Presentation A 52-year-old man has a dual-chamber pacemaker. He presents to the emergency department with recurrent syncope. The episodes are not associated with any prodromes. He receives the pacemaker because of complete heart block. His telemetry strip follows (Figure 1-A):

figure 1

17

Case 4  |  A 52-year-old Man with Pacemaker Malfunction and Syncope

Case Discussion What is the cause of the pacemaker’s behavior? The pacemaker is demonstrating the phenomenon of crosstalk. In crosstalk a paced atrial impulse in sensed in the ventricular chamber. Most frequently this occurs in pacemakers in the unipolar pacing mode. During unipolar pacing the vector of pacing is between the tip of the lead electrode and the pacemaker pulse generator (“can”). This leads to a wide area of electrical activity and increased propensity for sensing of the paced event in remote regions (such as the ventricle). This causes the pacemaker to interpret the atrial paced event as a ventricular sensed event. If the pacemaker is programmed to inhibit pacing in response to a sensed event (“demand pacing”), the ventricular lead will respond by inhibiting pacing. This can have catastrophic implications in a patient who is pacemaker dependent. Inhibition of ventricular pacing will lead to ventricular asystole. It should be noted that the crosstalk, by definition, occurs only in response to ventricular sensing of a paced atrial event. In the patient with delayed but intact AV nodal conduction, crosstalk is expected to lead to the observation of a prolonged paced AV interval beyond the programmed paced AV interval. This is because the paced atrial impulse has been sensed by the ventricular lead and has already led to the inhibition of ventricular pacing, or else ventricular pacing would have occurred at the end of the programmed paced AV interval. Intrinsic ventricular conduction is merely secondary (and very fortunate!). In the current tracing Figures 1A and B represent AV sequential pacing and appropriate capture. The third atrial paced beat (3) demonstrated is not followed by a ventricular paced artifact. Figure 1B demonstrates why: the atrial paced impulse is sensed by the ventricular lead (note S [sensed event] on the bottom ventricular marker channel). The sensed event leads to the inhibition of pacing. Also of note, this sensed event (from atrial pacing) initiates a new VA interval that actually pulls in the next atrial paced event (P-P 880 msec, all beats but 800 msec from the nontracked atrial spike to the next atrial spike). Also keep in mind that AV conduction block makes these findings much more readily appreciated.

18

Case 4  |  A 52-year-old Man with Pacemaker Malfunction and Syncope

Management The immediate management of crosstalk includes placement of a magnet that will revert the pacemaker to a “non-demand” pacing mode. No sensing or inhibition of pacing as a response to sensing will occur. A ventricular blanking period is programmed and defines the period of time after an atrial paced (but not sensed) event during which the ventricular lead is refractory to sensing. In this situation, electrical after potentials from atrial pacing will presumably no longer be present, minimizing the risk of the ventricular lead sensing atrial pacing. Programming to a bipolar pacing mode also minimizes the risk of crosstalk.

19

Case 5  D  evice Management after Atrioventricular Node Ablation  in an 82-year-old Man TEACHING POINTS • AV node ablation • Sudden death

• Device troubleshooting

Case Presentation An 82-year-old man has chronic atrial fibrillation with rapid ventricular conduction refractory to rate control medication. A decision is made to proceed with AV nodal ablation and pacing. After ablation of the AV node, the patient is left with an underlying heart rate of 30 beats/min and right bundle branch morphology.

figure 1 21

Case 5  |  Device Management after AV Node Ablation in an 82-year-old Man

A biventricular pacemaker is implanted in the VVIR pacing mode.

Case Discussion What specific precautions must be undertaken in the programming of the device? What are the potential complications of this procedure? Pacemaker implantation and AV nodal ablation leave most patients dependent on the pacemaker for cardiac activation. As such, any mechanical or other issues about integrity of the pacemaker can have catastrophic clinical consequences. Until recently, there has been a small but real association between sudden death and AV node ablation plus pacing. Although it may seem intuitive that the underlying causes of death are issues regarding the integrity of the pacing system, more recent data have suggested that the mechanism of death is polymorphic ventricular tachycardia. The mechanism for the phenomenon is unknown, but speculation surrounds the sudden decrease in heart rate (recall most patients undergoing ablation have had rapid ventricular rates). It is thought that the decrease in rate alters the repolarization properties of the ventricle such that there is increased predisposition to such arrhythmias. Death frequently occurs weeks after AV nodal ablation. It has been suggested that one means of eliminating the risk for polymorphic ventricular tachycardia is setting a relatively high lower rate limit for the pacemaker and gradually decreasing this setting. A common practice is to start at 80 or 90 beats/min and drop the lower rate by 10 beats/min on a monthly basis until a lower rate of 50 or 60 beats/min is achieved. Many individual practitioners have different approaches. Although controlled trials are lacking, anecdotal evidence suggests that this protocol can significantly decrease the incidence of sudden death after AV node ablation and pacing.

22

Case 6  A  75-year-old Woman with  a Pacemaker and Syncope during Prayer

TEACHING POINTS • Lead fracture • Device malfunction

• Oversensing

Case Presentation A 75-year-old woman presents with syncope. She had a pacemaker implanted because of sick sinus syndrome and progressive AV nodal conduction system disease. The episodes of syncope have no prodromes and last a few seconds. All episodes occur around 10 pm, as she prepares for bed. They are not associated with any changes in position. She does not take any diuretics or other blood pressure medications and denies positional light-headedness. A recent nuclear stress test showed normal left ventricular function without ischemia or scar. Examination is unremarkable. Blood pressure measurements fail to demonstrate any orthostatic changes. Electrocardiogram demonstrates sinus rhythm with a paced ventricular rhythm. Her pacemaker is a dual-chamber pacemaker with the following settings: DDDR, 50 to 120 beats/min Sensed AVI, 180 msec Paced AVI, 200 msec

23

Case 6  |  A 75-year-old Woman with a Pacemaker and Syncope

Interrogation demonstrates normal sensing, pacing thresholds, and battery function. The patient had an underlying ventricular rate of 30 beats/min.

Case Discussion What other investigations are appropriate at this time? Syncope in a patient with a pacemaker consists of the following differential diagnoses: 1.  Vasodepressor response: unlikely given the history and physical examination findings. 2.   Ventricular tachyarrhythmias: less likely given absence of anginal stress test, normal examination, and recent stress test. 3.  Noncardiac causes: possible in this case. The absence of any prodromes or postictal symptoms renders this less likely. 4.  Pacemaker malfunction: in a patient with known conduction system disease and syncope, bradycardia is the most likely cause of syncope. Detailed history is critical in elucidating a clear diagnosis. Further questioning of the patient revealed that all episodes had occurred while she was praying before turning in at night. During prayer, she would sit on the floor and clasp her hands forcefully, pressing them against each other. She would lose consciousness a few seconds later. What is the next appropriate step? Anytime that physical motion of the arms or shoulders is associated with syncope in a patient with a pacemaker, there is concern for a mechanical problem leading to pacemaker malfunction. In such situations, it is best to interrogate the pacemaker while the patient is repeating the same physical activities. Even if the history is not specific, performing provocative maneuvers such as flexion/extension of the arms and shoulders or compression of the palms against each other can elicit the responsible mechanism.

24

Case 6  |  A 75-year-old Woman with a Pacemaker and Syncope

In another case, during arm clasping, the following tracing was obtained:

figure 1

What does this tracing demonstrate? The tracing demonstrates high-frequency ventricular sensed events during arm clasping (asterisk). The cycle length of these events is too short to be physiological and likely represents noise artifact. Given that these events were associated with mechanical maneuvers, they likely represent insulation failure or lead fracture. In both cases, noise is sensed as physiological activity and leads to inhibition of pacing. In the pacemaker-dependent patient, this leads to syncope.

25

Case 7  A  54-year-old Man with Permanent Pacemaker Presents with Fever

TEACHING POINT • Device infection

Case Presentation A 54-year-old man with a history of heart failure and atrial fibrillation called the office complaining of fever and chills. He had undergone atrioventricular node ablation with pacemaker (PM) implantation 2 months prior. His cardiologist asked him to go to the emergency department. You were paged to see the patient. Pertinent medical history and clinical presentation He started having chills the day before admission. He also complained of pain and redness on his upper left chest on the area of the PM. He denied any other symptoms. His medical history included diabetes mellitus and myocardial infarction. He had coronary artery bypass surgery last year. His medications included insulin, atenolol, simvastatin, aspirin, enalapril, and warfarin. On examination, he had a temperature of 100.4˚F. The PM pocket was erythematous and tender to palpation but without fluctuation or signs of fluid collection. There was a 2/6 holosystolic murmur on the left sternal border and painful nodules on the second and third fingers of the left hand.

Case Discussion Differential diagnosis • PM pocket infection • PM infection with endocarditis

27

Case 7  |  A 54-year-old Man with Permanent Pacemaker Presents with Fever

Implantable cardiac device infection Background: • There are two main categories of device infections—(1) pocket infections: involve the subcutaneous pocket containing the device and the underlying leads but not the transvenous segment and (2) systemic infections: involve the transvenous portion of the lead, usually associated with bacteremia and/or endocarditis. • Based on the source of infection, it can be classified as primary (the device or pocket is the source of infection) or secondary (the leads are seeded from another source). Device infection can be associated with valvular infection, although it may remain uninfected in the setting of valvular endocarditis. The true incidence of device infection is unknown but has been reported to be in the range of 0.8% to 5.7%. Several factors are associated with device infection: recent manipulation of the device (eg, generator exchange), previous temporary pacing, diabetes mellitus, malignancy, operator inexperience, older patient, treatment with anticoagulants or glucocorticoids, previous fever, and early reintervention. Staphylococcus aureus and coagulase-negative staphylococci, often Staphylococcus epidermidis, cause more than two thirds of generator pocket infections and most device-related endocarditis. Infections within the first two weeks of implantation are mainly caused by S aureus. The most common organisms responsible for secondary infections is S aureus infections. Streptococci, Corynebacterium species, Propionibacterium acnes, gram- negative bacilli, and Candida have been reported to cause pocket infections and device-related endocarditis. Pocket infection commonly presents with fever and/or chills, general weakness, and other constitutional symptoms. Pocket infections can present with swelling, erythema, and pain at the site of the device in addition to systemic symptoms, particularly in the first few months after implantation. Inflammation over the site or erosion of the device through the skin are highly consistent with pocket infection. The positive culture of material aspirated from the inflamed site confirms the diagnosis.

28

Case 7  |  A 54-year-old Man with Permanent Pacemaker Presents with Fever

Transvenous electrode infection can effect intracardiac lead integrity and lead to right-sided endocarditis. Left-sided endocarditis is rare. The patient may present with sepsis syndrome and shock. The presentation is usually subacute. The common presenting symptoms are fever; chills; pulmonary involvement such as pneumonia, bronchitis, lung abscess, or embolism; tricuspid regurgitation; or mechanical stenosis from large vegetations. Epicardial electrode infection can present with signs of pericarditis or mediastinitis. Bacteremia is usually present. Up to 50% of patients with device-related endocarditis have evidence of vegetation on a valve, most commonly the tricuspid valve. By adding the pertinent pocket abnormalities and pulmonary emboli as clinical variables to the major Duke criteria, a clinical diagnosis of device-related endocarditis can be made in most of the confirmed cases. In the evaluation of patients with suspected device-related endocarditis and nondiagnostic transthoracic echocardiogram, information from a transesophageal echocardiogram should be used in the modified Duke criteria. Initiation of empiric antistaphylococcal antibiotics in case of suspected pocket or generator infection is recommended. Considering the high incidence of methicillin resistance S aureus and S epidermidis, initial therapy with vancomycin is reasonable. In the case of device-related endocarditis or pocket infection with bacteremia, empiric antibiotic therapy of endocarditis is recommended. The antibiotic regimen can be changed once the causative organism(s) is (are) identified in blood and/or wound cultures. In the current case, a transesophageal echocardiogram was ordered and revealed a vegetation on the right ventricular lead with no evidence of valvular endocarditis. The blood culture was positive for S epidermidis. Critical management questions   Would you explant the device, lead, or both?   What is the duration of antibiotic therapy? The device should be removed when there is pocket infection with or without associated bacteremia or lead infection, with or without endocarditis.

29

Case 7  |  A 54-year-old Man with Permanent Pacemaker Presents with Fever

Device explanation is also recommended for patients with the following: • S aureus bacteremia in the absence of an alternative source. • Bacteremia that persists or recurs with no alternative source despite appropriate antibiotic therapy. In the setting of infection limited to the pocket or subcutaneous tissue, it is recommended to continue antibiotics for 10 to 14 days after explantation of the infected system. The transition to oral antibiotics depends on the presence of systemic signs of infection, the local infection severity, and the urgency of implanting a new system. When and where to implant the new device? Until the initial infection is fully treated or controlled, reimplantation should not be attempted. In PM-dependent patients, temporary transvenous pacing should be continued until blood cultures are negative and endocardial infection has been controlled. This may require two weeks of parenteral therapy. However, in patients with pocket infections and no bacteremia or endocarditis, the new device can be implanted in a new location soon after explantation. The new device should be implanted at a different site from the explanted infected system, ideally on the opposite side. In the current case, after two weeks of therapy, the systemic symptoms resolved and repeat blood cultures were negative. The new PM was implanted and patient was discharged home on oral antibiotics. Besides the routine PM care recommendations, is antibiotic prophylaxis recommended? Transient bacteremia due to mucosal trauma rarely causes device infection. In the absence of another independent risk factor for endocarditis, antibiotic prophylaxis is not routinely recommended for patients with PMs or implantable cardioverter defibrillators. Some experts recommend antibiotic prophylaxis for procedures such as endoscopy, cystoscopy, or dental work in the first two months after implantation, before endothelialization is complete.

30

Case 8  A  56-year-old Man with Possible Device Malfunction

TEACHING POINTS • Ventricular safety pacing • Crosstalk

• Oversensing • Pacing inhibition

Case Presentation A 56-year-old man with a dual-chamber pacemaker is noted to have the following rhythm strip shortly after . surgery (Figure 1).

figure 1

31

Case 8  |  A 56-year-old Man with Possible Device Malfunction

Case Discussion What observation can you make regarding the pacemaker’s behavior? The first two beats demonstrate AV sequential pacing. Subsequently, five beats of a regular wide complex tachycardia is noted. The tachycardia is not preceded by any P waves and thus represents ventricular tachycardia (VT). After termination of the VT, AV sequential pacing resumes. Simultaneous with the onset of VT, the first of two pacing spikes are observed (asterisk). The first of the two spikes is exactly the same interval from the previous atrial spike (double asterisk) as the two previous atrial spikes were from each other (triple asterisk) (840 msec). This makes the first spike, simultaneous in onset with VT, likely an atrial spike. The following spike is therefore a ventricular pacing spike. Shortly after the first spike, and at an interval of about 100 msec, the ventricular spike is noted (asterisk). This is considerably shorter than the previous intervals (160 msec). This likely represents ventricular safety pacing (VSP). During VSP, if the ventricular lead senses any event within safety pacing interval, a ventricular pacing spike is delivered. The VSP is initiated shortly after every atrial pacing spike. Its purpose is to prevent crosstalk. Crosstalk refers to inhibition of ventricular pacing due to sensing of the atrial pacing spike by the ventricular lead. In other words, the ventricular lead erroneously assumes that the sensed atrial spike is ventricular activity. In a patient with AV block, such inhibition would lead to asystole. To avoid this situation, the ventricular lead is programmed to deliver a rapid pacing spike after sensing any electrical activity after atrial pacing. Of two situations, one will be operative: if the sensed event was atrial in origin, the ventricular pacing is appropriate. If the sensed event was truly ventricular, the fact that the ventricular pacing occurred rapidly assures that the ventricular spike will not be proarrhythmic (“spike on T wave”). In the current tracing, the atrial spike leads to the initiation of a safety pacing interval on the part of the ventricular lead. Because of the simultaneous onset of VT, the ventricular lead senses activity immediately after atrial pacing (during safety pacing interval). The response will be VSP 80 msec later. 32

Case 9  A  73-year-old Woman with Possible Pacemaker Noncapture

TEACHING POINTS • Threshold testing • Oversensing

• Noncapture

Case Presentation The tracing below was obtained from the monitor in a 73-year-old woman who had undergone dual-chamber pacemaker implantation one day before (Figure 1).

figure 1

The device settings were as follows: DDDR, 60 to 110 beats/min Sensed AVI, 180 msec Paced AVI, 200 msec

Case Discussion Is there evidence of device malfunction? If so, which of the lead(s) is/are involved? An initial review of the tracing shows what appears to be intrinsic P waves followed by a pause. Approximately 1.4 s later, an atrial pacing spike (asterisk) is observed. This appears to capture the atrium and conduct to

33

Case 9  |  A 73-year-old Woman with Possible Pacemaker Noncapture

the ventricle. This is then followed by P waves with intrinsic ventricular conduction. The absence of pacing after the last P wave before the 1.4-s pause suggests oversensing and inhibition of pacing by the atrial lead. Similarly, the absence of ventricular pacing during the same interval suggests ventricular lead oversensing. Also of note, the PR intervals after the pause (double asterisks) are longer than the programmed intervals, further suggesting ventricular oversensing, and pacing inhibition. Thus, on initial review, it may be concluded that neither lead is functioning appropriately. Closer inspection, however, demonstrates that the atrial rate after the pause is slower than that before the pause. In fact, what occurred in this case was that pacemaker thresholds were being checked the morning after implantation. During assessment of atrial pacing thresholds, the pacemaker is temporarily set to atrial pacing without ventricular tracking (AAI). Atrial pacing spikes are not observed because their output is just above the capture threshold. During the pause, the output is subthresholded and not observable on the rhythm strip. Immediately thereafter, the atrial output is increased to maximum with a lower rate limit of 60. This leads to a visible pacing spike followed by atrial capture. There is subsequent onset of spontaneous atrial activity (note the rate is slower than that before the pause) with intrinsic conduction. This more rapid rate of atrial pacing during threshold testing had led to overdrive suppression in this patient with severe sick sinus syndrome. Atrial noncapture with amplitude decrement (part of the automatic threshold test) manifested the sinus pause. A similar observation had been made during an electrophysiology study. Take-home points are as follows: 1.   Rhythm strips are not reliable markers of pacing spikes. 2.  If it appears that there are multiple device malfunctions (in this case, oversensing of both atrial and ventricular leads), it is best first to investigate the scenarios under which the observations were made.

34

Case 10  Multiple Pacing Artifacts in a Critically Ill Patient

TEACHING POINTS • Device-device interactions • Pacemaker-dependent patients

• Undersensing • Noncapture

Case Presentation The following tracing was obtained from a 69-year-old man in the intensive care unit (Figure 1). The patient has a history of atrial fibrillation and sick sinus syndrome for which he received a single-chamber atrial (AAIR) pacemaker.

figure 1

He is admitted with fevers, chills, and syncope. Blood cultures were positive for gram-positive cocci.

Case Discussion What observations can be made? Three immediate observations can be made: (1) the underlying rhythm appears to be either atrial flutter or fibrillation (arrows), (2) the ventricular

35

Case 10  |  Multiple Pacing Artifacts in a Critically Ill Patient

rhythm is paced at a regular interval (V), and (3) two groups of pacing spikes can be observed (V for Vpace and A for Apace). One of those spikes is the ventricular lead pacing at regular intervals (star). The other is the atrial lead, which does not appear to be capturing the atrium. The atrial lead is pacing because it cannot sense the atrial fibrillation or flutter activity. However, it cannot capture the atrium because it is being constantly activated by the arrhythmia. The ventricular lead is, in fact, a temporary pacemaker placed because the patient has bacterial endocarditis, which has destroyed the AV conduction system creating AV block and causing syncope. Thus, the patient has two independent pacing systems.

36

Case 11  A  74-year-old Man with  Dual-chamber Pacemaker  and Shortness of Breath

TEACHING POINTS • Oversensing • Mode switching

• Device malfunction

Case Presentation The following tracing was obtained from a 74-year-old man with a dualchamber pacemaker (Figure 1).

figure 1

The patient had a history of atrial arrhythmias and sinus node dysfunction. He presented with onset of increased fatigue and shortness of breath.

37

Case 11  |  A 74-year-old Man with Dual-chamber Pacemaker

Case Discussion Based on review of the tracing (Figure 1), are there any observations that may explain the patient’s symptoms? Two channels are shown. The top channel is the atrial channel; the bottom is the ventricular channel. Below the channels are the electrogram markers, which represent the pacemaker’s interpretation of the raw signals demonstrated on each channel. The first beat demonstrates an atrial sensed (AS) event (star) followed by a ventricular paced event (VP) (double star). Detailed attention to the atrial signals reveals that there is a signal after the first AS signal. This second signal is slightly later than the ventricular paced signal and reflects either far-field sensing of the ventricular event by the atrial lead or retrograde ventriculoatrial conduction via the AV node. Although the signal is seen in the atrial channel, we know that it is not interpreted as an atrial event because there is no corresponding marker on the marker channel. The subsequent beats, however, demonstrate this second component to be interpreted by the pacemaker with the following marker: (AS). This universally can be interpreted as the pacemaker having sensed an atrial event but that it has chosen to ignore it because it is in a refractory period. The latter is demonstrated by surrounding the AS by parentheses. Although the device will “ignore” such AS events for the purpose of initiating an AV timing interval, it will not do so for the purposes of detecting (or “declaring”) an atrial tachyarrhythmia. As such, shortly after sensing the second atrial component on 3 successive beats, the device declares initiation of atrial arrhythmia (ATR-FB). The device then changes its pacing mode from a tracking mode (ventricular pacing in response to atrial sensing) to a nontracking mode wherein ventricular pacing is independent of AS events. This is referred to as mode switching and serves the purpose of avoiding rapid tracking of nonphysiological atrial arrhythmias. It can be appreciated the after ATR-FB, ventricular pacing becomes independent of AS events. Detailed analysis of the penultimate (triple star) beat also clarifies the etiology of the second component of the atrial

38

Case 11  |  A 74-year-old Male with Dual-chamber Pacemaker

electrogram. Note that after mode switching, the second component is always preceded by ventricular pacing. This, as noted above, may indicate retrograde VA nodal conduction or far-field sensing by the atrial channel. In the penultimate beat, V pacing is followed by the atrial signal, which is immediately followed by an AS event. We know that two successive atrial events cannot occur so rapidly as atrial refractoriness is expected to occur. This then indicates that the second component is indeed far-field sensing of a true ventricular event by the atrial lead. Finally, with regard to the patient’s symptoms, it can be appreciated that once ATR-FB is declared, there is no further AV synchrony, as the pacemaker is in a nontracking mode. Although in reality, there are no atrial arrhythmias, the loss of AV synchrony can lead to loss of the physiological “atrial kick” and, in the case of this patient, multiple symptoms.

39

Case 12  Ventricular Tachycardia in a Patient with a Dual-chamber Pacemaker

TEACHING POINTS • Noise • Oversensing

• Pacemaker malfunction

Case Presentation A 62-year-old female with sick sinus syndrome underwent radiofrequency ablation of atrial fibrillation. She has a dual-chamber pacemaker. The morning after her procedure, you are called to assess a wide complex tachycardia noted on telemetry. The tracing below is obtained (Figure 1):

figure 1

41

Case 12  |  Ventricular Tachycardia in a Patient

Case Discussion What is the underlying rhythm? Initial evaluation of the tracing demonstrated a wide complex tachycardia. This is concerning for ventricular tachycardia (VT). Closer inspection, however, demonstrates that the first impulse is an atrial paced beat with intrinsic ventricular conduction (star and double star). The atrial pacing artifact is followed by a captured P wave and intrinsic QRS. In the midst of the wide complex beats, these atrial impulses can be seen with regular frequency (arrows). This suggests that atrial pacing is occurring at a regular interval despite what, on initial glance, appears to be possible VT. Ventricular pacing is also possible but less likely as the morphology of the spikes looks consistent. Further analysis actually reveals what are most likely intrinsically conducted QRS complexes, which have been distorted. These also occur at regular intervals after each atrial paced beat, again making ventricular spikes less likely. These intervals approximate the spike-QRS interval of the first beat. A pacemaker is very unlikely to continue atrial pacing during VT. One circumstance in which this could occur would be undersensing of VT. In that situation, however, one would expect to see ventricular pacing spikes. This is not seen in the current tracing. Even more suggestive against a diagnosis of VT are the discernible conducted QRS complexes. Put together, these observations suggest that the wide complex tachycardia is likely an artifact.

Features that are suggestive of noise in a patient with a pacemaker and wide complex tachycardia are as follows: • • • •

42

Absence of symptoms Regular pacing spikes or QRS morphology Source of ambient noise or interference History of Parkinson disease

Case 12  |  Ventricular Tachycardia in a Patient

The differential diagnosis of a wide complex tachycardia in patient with a pacemaker includes the following: • • • •

VT Inappropriate ventricular pacing Artifact Supraventricular tachycardia (SVT) with aberrancy

Interrogation of the device confirmed the absence of any VT during the episode with consistent atrial pacing and intrinsic ventricular conduction. Although ambient noise can create a high-frequency (often 60 Hz) artifact on telemetry leads, so can movement disorders such as Parkinson disease. Motion on the part of the patient can also lead to distortion of the signals. Surface lead movement or dislodgement can cause artifact but is unlikely in this case, as the intrinsic cardiac activity remained recorded during the course of artifact recording.

43

Case 13  Ventricular TachycardiaDetected during Pacemaker Interrogation

TEACHING POINTS • High-rate episodes • Ventricular tachycardia

• Marker channel analysis

Case Presentation You are informed by the pacemaker nurse that one of your patients has had multiple ventricular high-rate episodes detected on her pacemaker. The device is a dual-chamber Medtronic pacemaker and is programmed to detect ventricular high-rate episodes (VHREs) at a rate of 180 beats/min at 5 beats. The longest and fastest episodes’ electrograms are stored. The tracing below is representative (Figure 1):

figure 1

45

Case 13  |  VT Detected during Pacemaker Interrogation

Case Discussion What further evaluation may be considered? What is the most likely diagnosis? The tracings show the atrial and ventricular channels. Analysis shows an abrupt increase in the ventricular rate. It is challenging, if not impossible, to assess changes in the ventricular morphology of the rapid beats. The atrial channel demonstrates a rate slower than the ventricle. This feature is suggestive of ventricular tachycardia (VT), and the pacemaker has accordingly placed this in the VHRE bin. There are, however, other causes for a V rate greater than A. These include junctional or atrioventricular nodal reentrant tachycardias with retrograde atrial block or ventricular channel noise. Closer inspection demonstrates that at the onset of the VHRE, and immediately preceding it, one can observe atrial premature activity (star), which is followed by the first ventricular premature beat. Another atrial premature impulse can also be seen, followed by a train of ventricular beats. During the remaining course of the ventricular run, only one more atrial impulse is seen. Once the VHRE is completed, atrial and ventricular sensed events (presumably sinus) follow. One observation in this tracing is that the first premature beat is of atrial origin and is followed by a ventricular impulse. This may lead to the interpretation that the entire episode is of atrial origin. If this is the case, then one has to explain why there is no atrial sensed event for most of the VHRE. A number of explanations can be operative: first, if this was a brief run of atrial tachycardia (AT) with conduction to the ventricle, it may have led to atrial undersensing of the AT simply because these are of a different origin. The second explanation may be that the subsequent atrial beats were in atrial blanking or refractory periods. That is, each ventricular conducted beats led to subsequent atrial blanking period, which did not allow detection of atrial activity. If this were the case, however, we would have expected to seen AR, a marker for atrial events sensed in the refractory period. Except for the second premature atrial beat, we do not see any such electrograms. This argues against AT with refractory or

46

Case 13  |  VT Detected during Pacemaker Interrogation

blanked atrial signals. It is theoretically possible that all atrial events were in the blanking period, which is not sensed or annotated. The blanking period, however, was very short (50 msec in this case), making blanking of all atrial events (if this were an atrial arrhythmia with rapid ventricular conduction) unlikely. The second premature atrial event may represent retrograde conduction of the first beat of VT (double star). In this case, the first atrial premature beat coincidentally preceded the onset of VT. Also note that the first sinus beat after the tachycardia occurs at a relatively close coupling interval. If this were an AT, some degree of sinus node suppression would be expected after the termination of the tachycardia. This is not observed. Another trick is to observe previous atrial high-rate episodes. If these are of the same rate as the VHRE, the conclusion that VHRE was of atrial origin is further supported. Finally, and obviously, if electrograms were available, their review would have facilitated a diagnosis. In this pacemaker model, however, electrograms were not available. The tracing is most likely demonstrating VT.

47

Case 14  Intermittent Atrial Pacing in a 62-year-old Man with  a Dual-chamber Pacemaker

TEACHING POINTS • Mode switching • Undersensing

• Atrial fibrillation • AV block

Case Presentation The following ECG was obtained form a patient with a dual chamber pacemaker (DDD, 90 beats/min; AV interval 300 msec).

figure 1

49

Case 14  |  Intermittent Atrial Pacing in a 62-year-old Man

Case Discussion What observations can be made? The rhythm is ventricular paced at a rate of 90 beats/min. The first half of the electrocardiogram shows ventricular pacing. The second half demonstrates AV sequential pacing. The ventricular rate is unchanged. Close inspection demonstrates the absence of P-waves throughout the tracing. Importantly, there appears to be no change in P-wave morphology, inasmuch as there is no discernable P wave with or without atrial pacing. When atrial pacing is present, the A-V interval is always timed out. The patient is in atrial fibrillation (AF) with complete heart block. The first half of the tracing demonstrates sensing of AF by the atrial lead. The device goes into a mode switch (nontracking mode). In the second half, the atrial lead undersenses the fibrillatory atrial impulses and interprets the rhythm as atrial standstill. Thus, atrial pacing commences. This obviously does not alter the atrial rhythm of AF. Because the patient has AV block, the AV interval times out. Ventricular pacing ensues. This has no significant effect on the ventricular rhythm. Long-term consequences include premature depletion due to atrial pacing. The problem can be addressed by increasing the atrial lead sensitivity.

50

Case 15  Tachycardia during Pacing

TEACHING POINTS • Cardiac arrhythmias in patients with pacemakers

• Pacemaker lead malfunction

Case Presentation The rhythm strip on the next page was obtained from a patient with a dual-chamber (DDDR, 60–12) pacemaker.

Case Discussion What observation can be made? What does this suggest about the nature of the tachycardia? The rhythm strip initially demonstrates sinus rhythm with intrinsic AV conduction. There is appropriate sensing in the atrium and ventricle as manifested by the inhibition of pacing in both chambers. Subsequently, an atrial pacing spike captures the atrium and conducts to the ventricle (arrow). We know that atrial capture occurs because the atrial spike leads to a premature P wave. The reason for the initiation of atrial pacing most likely is atrial undersensing, as the interval between the first atrial pacing spike and the preceding P wave is too short for the atrial lower rate interval to time out (intrinsic P wave to the next spike is a 620-msec [97 beats/min] interval). Subsequently, and at a regular interval of 1000 msec (the lower rate limit of the pacemaker), atrial pacing occurs. Some of these do not capture the atrium. Before deciding if we are dealing with absence of atrial capture, it should be noted that intrinsic P waves have no effect on the atrial spikes. The P waves do not lead to atrial inhibition. Thus, one definite problem

51

Case 15  |  Tachycardia during Pacing

figure 1

52

Case 15  |  Tachycardia during Pacing

is atrial undersensing. Although it is true that the second atrial spike does not capture the atrium, it should be noted that a P wave immediately preceded it and that the atrium was likely refractory. The third spike does capture the atrium. The fourth spike occurs immediately after the P wave, and thus, the atrium was again refractory. The fifth spike, importantly, also captures the atrium. This is followed by a regular narrow complex tachycardia. Inverted P waves can be observed immediately after each QRS (asterisks). All ventricular pacing is inhibited. The fifth atrial spike acted as a premature atrial contraction and led to the onset of a supraventricular tachycardia, likely AV nodal reentrant tachycardia. The absence of A pacing during the SVT suggests that the P waves during the tachycardia were sensed by the pacemaker. Intermittent loss of atrial sensing eventually led to a captured atrial beat (fifth spike), which acted as a premature atrial impulse (PAC) to induce SVT. During SVT, the atrial electrograms were sensed with appropriate inhibition of further pacing.

53

Case 16  O  nset of Ventricular Tachycardia during Pacemaker Threshold Testing

TEACHING POINTS • PMT • Atrial undersensing

• Troubleshooting

Case Presentation The following tracing was obtained during threshold testing (Figure 1). You are urgently called for onset of VT during threshold testing. The setting is DDD at 90 beats/min.

figure 1 55

Case 16  |  Onset of VT during Pacemaker Threshold Testing

Case Discussion What phenomenon is observed? The top panel shows AV sequential pacing with capture of both chambers. In the middle panel, only the second atrial pacing spike captures the atrium (note absence of P waves, best seen on the bottom row of the middle panel, and corresponding retrograde P waves, manifested as a notch in the T wave of the top row of the middle panel). After the first atrial spike in the middle panel, there is absence of further atrial spikes (arrow). Retrograde P waves (star), which could be seen earlier with each noncaptured atrial spike (double stars), are now sensed and tracked. In the bottom panel, the third retrograde P wave is no longer tracked and the tachycardia terminates. The absence of atrial capture is one of the causes of pacemaker-mediated tachycardia as subsequent ventricular paced impulses can conduct retrogradely via the AV node (because no atrial capture occurred, there was no penetration of the AV node and it can accommodate retrograde conduction). Once enough delay in retrograde conduction occurs (after the sixth ventricular paced beat in the middle panel), the P waves fall outside of the postventricular atrial refractory period: it is sensed, tracked to the ventricle (because the AV node was just activated retrogradely, it is unlikely to accommodate antegrade conduction), by which time the node has recovered and able to assume retrograde conduction. A pacemaker mediated tachycardia (PMT) circuit is thus initiated and sustained. Termination of PMT is due to an algorithm that prevents tracking of the atrial impulse after a predetermined number of Asense-Vpace events. This interrupts the “circuit” of PMT, terminating the tachycardia. The abrupt onset of PMT should alert the physician to the possibility of atrial lead malfunction. It should be kept in mind that both atrial noncapture (as described above) and atrial undersensing can lead to PMT. In the case of atrial undersensing, a P-wave is not tracked (because it is not sensed). Because a P wave is not sensed, the device follows with an atrial spike that usually does

56

Case 16  |  Onset of VT during Pacemaker Threshold Testing

not capture (because the atrium is refractory because it was just activated by the intrinsic and undersensed P wave). Atrial noncapture follows, and the remainder of the mechanism of PMT initiation is as described above. Management in this case is to make the atrial lead more sensitive by reprogramming or repositioning.

57

Case 17  Dual-chamber Pacemakerwith Possible Lead Malfunction

TEACHING POINT • Managed ventricular pacing

Case Presentation The following tracings were recorded during routine telemetry (Figure 1). The patient had received a dual-chamber Medtronic pacemaker for sinus node dysfunction and intermittent AV block.

figure 1

59

Case 17  |  Dual-chamber Pacemaker with Possible Lead Malfunction

Case Discussion What phenomenon is observed? The top panel demonstrates AV sequential pacing for the first seven sequences. There is appropriate capture of the atrium and ventricle without evidence of atrial or ventricular noncapture. The eighth atrial paced impulse is not followed by ventricular pacing. The ninth paced atrial impulse (last impulse) is followed by intrinsic conduction. The bottom panel, in essence, starts with the penultimate atrial paced impulse on the top panel. The first atrial impulse is nonconducted and not tracked. This may lead to the conclusion of ventricular oversensing by the ventricular lead. This is because the ventricular lead has inhibited pacing even though there is no ventricular activity. The second atrial impulse is conducted via the native conduction system. Then, intrinsic sinus activity and native conduction take over. The first intrinsic QRS in the second panel is narrower than the subsequent ones (which appear to have incomplete right bundle branch morphology). In this case, intrinsic conduction occurs with appropriate inhibition of ventricular pacing. This rules out ventricular undersensing. The narrow initial QRS followed by slightly wider conducted impulses likely represents increased time for bundle branch recovery in the first conducted beat. Although it may appear that there is device malfunction (specifically ventricular oversensing with inhibition of ventricular pacing), this algorithm is part of the managed ventricular pacing algorithm used in dualchamber devices (implantable cardioverter defibrillators and pacemakers). Its underlying purpose is to minimize the amount of right ventricular pacing as it is thought that right ventricular pacing can be deleterious to myocardial function. The algorithm, in essence, gives the AV node every chance to conduct by transiently acting in a nontracking mode. In this situation, a paced or sensed atrial impulse does not initiate an AV timing interval. This allows maximum opportunity for the impulse to conduct to the ventricle. A blocked atrial beat is tolerated so long as the subsequent beat conducts. These tracings demonstrate the latter finding, wherein the device allows native ventricular conduction even if it requires allowing a blocked atrial impulse for a single beat. In this case, this allowed recovery of conduction.

60

Case 18  Inflammatory Reaction on Device Site

TEACHING POINTS • Hypersensitivity to device composites

• Management of device allergy

Case Presentation A 75-year-old white male was evaluated for swelling and redness over his pacemaker (PM) site. He was diagnosed with third-degree AV block and a permanent PM was implanted on his upper left chest two months earlier. He noted redness, pain, and swelling in the area of the PM three days before presentation. He denied any fever or chills but admitted to general weakness. Medical history was remarkable for diabetes, coronary heart disease, hyperlipidemia, and hypertension. He denied any allergy. He was a smoker and consumed alcohol socially. Medications include atorvastatin, lopressor, aspirin, and pioglitazone. Examination was remarkable for a low-grade fever, erythema, swelling, and mild tenderness over the PM site (Figure 1).

figure 1

61

Case 18  |  Inflammatory Reaction on Device Site

Case Discussion The differential diagnoses for his presentation are mainly device infection and rarely allergic reaction to metal. What to do next? The patient was hospitalized with possible diagnosis of PM pocket infection. The PM generator was removed, a temporary PM was placed, and he was started on antibiotics. The blood work, including complete blood count and electrolytes, were unremarkable. Transeshophageal echocardiogram was unremarkable for any vegetation. Two sets of blood cultures and the pocket swab culture remained negative after four days. The erythema and swelling improved. What to do next? • Implant a new generator on the right side and discharge him on antibiotics. or • Perform a skin patch test for metal allergy. Infection is the most common cause of inflammation at implantable cardiac device sites. Although infection should be thoroughly investigated, an allergic reaction should be kept in mind (see Case 7, page 27). Previous studies have reported negative cultures in groups of patients with signs and symptoms of device infection, which are thought to be due to previous antibiotic use. Hypersensitivity and allergic reactions to the metallic composite of implantable heart rhythm devices (IHRDs), including implantable cardiac defibrillators and pacemakers, are rare complications of IHRD implantation. Patients can present with local or systemic signs and symptoms of inflammation and /or device malfunction. Different brands of IHRDs contain different composites, and these can be obtained from their vendors. Mainly, IHRD generators are covered with a titanium capsule. The header, where leads are attached to the generator, has two components: polymethylmethacrylate (the glassy part) and

62

Case 18  |  Inflammatory Reaction on Device Site

silicone rubber. The leads are composed of conductor wires and pacing or shock electrodes. The conducting wires are mainly composed of an alloy of Ni, Co, Cr, and Mo (MP35N ) or MP35N with a silver core for highcurrent use. Pacing electrodes are made of platinum, platinuiridium, or tantalum with platinum coating. Some defibrillators may have a titanium shock coil, although these are not common. Allergic reactions to IHRDs commonly present with dermatitis and pain over the implantation site from two days to two years after implantation. Rarely, it can also present as generalized puritis. Any of the metallic components can be the culprit. For instance, titanium allergy can be diagnosed by patch testing, lymphocyte proliferation tests, or intradermal test with serum contained titanium. The same methods can be used to test allergy to other components. Management includes local steroid for mild dermatitis or removal of the device and implantation a new one without the causal composite. The successful use of gold- or silicone-coated devices and leads in prevention of sensitivity reaction has been reported previously. Custom-made IHRDs without allergen composite can be used for these patients.

63

Implantable Cardiac Defibrillators

Implantable Cardiac Defibrillator Teaching Points (ICD) Arrhythmia detection algorithms, ICD Case 2, ICD Case 8 Arrhythmias diagnosis algorithm, ICD Case 5 Atrial high rate events, ICD Case 6 Atrial tachycarrhythmias, ICD Case 12 Audible alerts, ICD Case 14 Blanking periods, ICD Case 1 Detection algorithms, ICD Case 11 Device Diagnostics, ICD Case 9 Device Troubleshooting, ICD Case 8, ICD Case 14 Double counting, ICD Case 1, ICD Case 8, ICD Case 9 Fusion, ICD Case 3 Inappropriate ICD shocks, ICD Case 2, ICD Case 4, ICD Case 9 Lead fracture, ICD Case 4 Lead malfunction, ICD Case 9 Mode switching, ICD Case 6, ICD Case 10, ICD Case 12, PM Case 11, PM Case 14

Morphology, ICD Case 11 Noise, ICD Case 4 Noncapture, ICD Case 3 Oversensing, ICD Case 1 Pacemaker-mediated tachycardia, ICD Case 13 Pacing inhibition, ICD Case 10 Pacing timing intervals, ICD Case 6 Pseudofusion, ICD Case 3 Sensing, ICD Case 8, ICD Case 11, ICD Case 12 Sinus capture, ICD Case 10 Total atrial refractory period (TARP), ICD Case 7 Tracking, ICD Case 12 T-wave oversensing, ICD Case 1, ICD Case 8, ICD Case 9 Upper rate behavior, ICD Case 12 Variable atrial pacing, ICD Case 7 “Wobble” phenomenon, ICD Case 5

67

Case 1  Exercise-induced Implantable Cardioverter Defibrillator Shocks

TEACHING POINTS • Oversensing • T-wave oversensing

• Double counting • Blanking periods

Case Presentation A 54-year-old man with a history of prior myocardial infarction presents with multiple ICD shocks, each occurring shortly after jogging. The tracing below was obtained during interrogation of his device (Figure 1).

figure 1 

Case Discussion What observations can be made? The top channel (atrial) is followed by a far-field event that is not sensed. The ventricular channel, however, demonstrates a ventricular paced event (no atrial signal is discernable) followed by a ventricular sensed event. The causes for a sensed event after a paced event include the following:

69

Case 1  |  Exercise-induced ICD Shocks

• • • •

Failure to capture and intrinsic conduction comes through. Premature ventricular contractions (PVCs). Intermyocardial or intrafascicular reentrant echo beats Capture latency giving rise to local myocardial capture beyond the ventricular blanking period • Far-field signal from trans-septal myocardial activation It should be noted that it is normal and expected for sensing to occur immediately after a paced beat, as tissue depolarization can last well beyond the duration of the stimulus artifact. To avoid this, devices have a blanking period programmed. This period is usually not programmable and can last anywhere between 20 and 200 msec for Medtronic devices, 125 to 157 msec for St. Jude devices (programmable), and 20 to 200 msec for Boston Scientific devices (programmable). Is this a second ventricular event, independent from the ventricular pacing? Two features make this unlikely: first, there is a fixed interval between the paced and the sensed event. Such fixed coupling may imply that the second (sensed) event is dependent on the paced event. Although a reentrant beat, for example, may be associated with a paced beat, this is somewhat unlikely in the current situation given the next observation. The very close coupling interval between the paced and the sensed events makes it physiologically less likely for the sensed event to be an independent impulse and strongly implies “double counting” of the paced beat. What has occurred is that the paced impulse has encountered delay in conduction to such an extent that ventricular myocardium is still being activated after the ventricular paced refractory period is over. The ventricular channel detects and senses this delayed activation as a ventricular sensed event. Could this be T-wave oversensing? This is not likely because the interval between the ventricular paced and sensed events is too short for the T wave to have occurred. Shortly after the first tracing, the following tracing is obtained (Figure 2).

70

Case 1  |  Exercise-induced ICD Shocks

figure 2 

What observations can be made? We now see that at a slower atrial sensed rate, ventricular paced events are not associated with a ventricular sensed event after ventricular pacing. A likely explanation is that as the rate slows down, the delay in ventricular conduction of the ventricular paced impulse lessens. At the faster rate, either the blanking period is shorter or the ventricular electrograms are occurring later. As discussed above, the absence of such a delay eliminates the substrate for double counting. Furthermore, at the faster rate, the first, third, and fifth beats on the atrial channel sense the ventricular paced beat. This may be because the delay in ventricular activation exceeds the postventricular atrial blanking period, a timing used in the atrial channel so that the paced ventricular beat is not sensed in the atrium (similar to the ventricular blanking period). How would we manage this situation? The most likely cause of this observation must be analyzed: based on the surface tracing, we know that ventricular capture occurs and that reentrant beats or PVCs do not follow. Sensing occurs before the T wave, ruling out T-wave oversensing. The regularity of the sensed event rules out noise. Thus, the most likely cause is local ventricular latency leading to sensing beyond the blanking period. A management strategy would be to

71

Case 1  |  Exercise-induced ICD Shocks

increase the postpacing refractory period so that delayed activation due to pacing would be less likely to be sensed. In the case of T-wave oversensing, double counting, or noise, another option would be to decrease the sensitivity of the ventricular lead. In this situation, one can increase the value, telling the pacemaker to ignore any signals smaller than the cutoff. By turning up the sensitivity parameter value, the device has become less sensitive because it is being instructed to ignore more signals. If you see only people taller than 6 ft 2 in (higher sensitivity setting), you will see less people than if you see people taller than 5 ft (lower sensitivity setting). Decreasing the device’s sensitivity, however, may increase the probability of undersensing ventricular arrhythmias and should be done cautiously.

72

Case 2  Multiple Implantable Cardioverter Defibrillator Shocks in a 45-year-old Man TEACHING POINTS • Inappropriate ICD shocks

• Arrhythmia detection algorithms

Case Presentation A 45-year-old man presents with 13 ICD shocks, all of which occurred within a 5-min period. The following tracings are obtained (Figure 1):

Figure 1  Tracings showing irregular V. 73

Case 2  |  Multiple ICD Shocks in a 45 year-old Man

Case Discussion What observations can be made? The tracings demonstrate irregular ventricular activity. The morphology of the tracings remains unchanged during slower versus more rapid ventricular activity. These findings are highly suggestive of atrial fibrillation (AF) with rapid ventricular conduction as the etiology for the rapid ventricular activity. Because the conduction exceeds the cutoff for ventricular fibrillation detection, the device will ignore any other features (so-called discriminators, discussed below) used in determining whether to “declare” an episode. In this case, the device declares (erroneously) ventricular fibrillation and delivers a shock. What is more telling, however, is the rate of activation after the shock. Very rapid conduction of AF can be observed. In general, the more rapidly AF conducts, the less variability one observes between the beats. As such, one of the important discriminating features is rendered less useful. Furthermore, the 9th and 18th beats (star) are aberrantly conducted, rendering another discriminator (morphology). Three common discriminators are commonly used to differentiate rapid ventricular conduction of supraventricular arrhythmias from true ventricular arrhythmia: • Morphology: an algorithm performs a morphological analysis of the in­tracardiac electrograms. Similarity to baseline values argues against ventricular tachycardia (VT). • Stability: variability in beat-to-beat intervals suggests atrial fibrillation with irregular ventricular conduction. • Onset: sinus tachycardia usually presents with a gradual increase in heart rate. Thus, an onset that is not abrupt (gradual increase in rate) suggests sinus tachycardia. There are two very important observations from this case: first, a shock is a very traumatic experience. There is a tremendous adrenergic response that leads, very often, to more rapid conduction of AF. This then leads to further shocks, followed by even more heightened anxiety, thus initiating a positive feedback loop that can lead to multiple ICD shocks. Second, when a patient complains of multiple shocks over a very brief period of time, there should be a high index of suspicion for inappropriate shocks. 74

Case 3  I nterrogation of a Dual-chamber Implantable Cardioverter Defibrillator TEACHING POINTS • Fusion • Pseudofusion

• Noncapture

Case Presentation The following tracing is obtained during routine interrogation of a dualchamber ICD (Figure 1).

FIGURE 1  75

Case 3  |  Interrogation of a Dual-chamber ICD

Case Discussion What observations can be made? The top line shows the surface ECG, the middle channel is the markers, and the bottom channel shows the atrial electrogram. “A” reflects atrial pacing, “R” is intrinsic ventricular activity, and “V” is a paced ventricular event. The first beat shows an atrial paced event followed by what the device deems to be a ventricular paced event. Indeed, the ventricular pacing artifact can be observed on the atrial channel (asterisk). The surface QRS is different than the fourth, fifth, and sixth QRS morphologies, none of which are paced. Thus, this beat is, at least in part, paced. In the St. Jude devices, a paced impulse is noted by a V on the channel marker (A for the atrium), whereas an intrinsic ventricular beat is annotated by R (P for the atrium). Very importantly, the QRS morphology of the second beat is distinct from the first (paced) and identical to the last three impulses. We know that the last three impulses are not paced (the marker channels show R). Thus, the second beat, although marked as paced (V), is not paced. This phenomenon, wherein a paced stimulus never captures the chamber because it is preempted by intrinsic activation, is labeled pseudofusion (see below). The third beat, however, shows an atrial paced event followed by a ventricular sensed event. The device has not delivered a pacing artifact, nor can one be seen on the atrial channel (as a far-field observation from the ventricular chamber). The QRS morphology, however, looks unchanged. The same phenomenon repeats itself in the fourth beat. It can be concluded now that the ventricular chamber was never paced, as the QRS morphology between the paced and the nonpaced beats looks identical. Do these observations signify ventricular noncapture? Not necessarily. What is more likely occurring (although ventricular noncapture is still a possibility) is pseudofusion. In pseudofusion, a paced impulse is delivered, but this paced impulse does not contribute to myocardial activation. More often than not, the reason is simultaneous native

76

Case 3  |  Interrogation of a Dual-chamber ICD

conduction, which precludes the activation of that chamber by the pacing stimulus. In the current tracing, it is likely that the ventricular activation of the first two beats were delayed just long enough for ventricular pacing to be delivered (by virtue of reaching the lower rate limit interval of the pacemaker). Antrioventricular (AV) conduction, however, preempted the ventricular paced artifact, and conduction occurred such that ventricular activation was not contributed to by the paced impulse. The distinction between paced, fused (true combination of pacing and intrinsic activation), and pseudofusion has two clinical implications: in cardiac resynchronization therapy, the device interprets a pseudofused beat as a paced beat. This may erroneously overestimate the frequency of resynchronization. Pseudofusion is, in essence, pacing output without effective capture. This can lead to a drain on the battery. Furthermore, both tracings demonstrate progressive shortening of AV intervals. The last three tracings show R waves that occur immediately after or simultaneously with the paced atrial impulse. This suggests that the R waves are not conducted. A rapid junctional rhythm may be operative. As a final point, we have extended the tracing to show the impulse before the tracing above (arrow). Here, we can see atrial and ventricular pacing with a paced QRS morphology. This reinforces that pseudofusion, not ventricular noncapture, was operative above.

77

Case 4  Long-QT Syndrome and Multiple Implantable Cardioverter Defibrillator Shocks

TEACHING POINTS • Inappropriate ICD shocks

• Lead fracture • Noise

Case Presentation A 26-year-old with a single-chamber ICD because of inherited long-QT syndrome has 22 ICD shocks while dancing at a nightclub. At the ER the following tracings are obtained (Figure 1):

FIGURE 1  From top to bottom: surface electrocardiogram (ECG),

electrogram, and electrogram marker channel. The electrograms are the far-field right ventricular lead (tip to coil).

79

Case 4  |  Long-QT Syndrome and Multiple ICD Shocks

Case Discussion What caused the shocks? The surface ECG demonstrates sinus rhythm with intrinsic conduction. The first two and the fifth beats are aberrantly conducted premature atrial contractions. This is reflected in the variation in the morphology of the corresponding intracardiac electrograms. The electrograms, however, demonstrate multiple signals not associated with any activity on the surface ECG. Importantly, the intervals of some of these signals (100 msec between the seventh and eighth signals on the marker channel) are at nonphysiological intervals. It is close to impossible to have ventricular recovery at such brief intervals. Some of the signals, therefore, do not reflect myocardial activity. If one traces each QRS from the surface ECG, regular activity associated with each QRS can be traced for both electrogram channels. These obviously represent true ventricular activation. The remaining do not. What are the other sources of sensed activity as defined by the electrograms? T-wave oversensing is possible but is not the case here, as many of the deflections do not coincide with the surface T waves. One would expect a regular interval between the actual sensed QRS and the oversensed T wave. This is not the case in the current tracing. Another possibility is oversensing due to potentially latent intrinsic depolarization (“QRS oversensing”). One of the causes may be a short postventricular blanking period. In this case, one would expect all “extra” electrograms to be associated with the surface QRS. This is not the case. This leaves two other diagnostic possibilities: sensing of external noise (amplifiers in a nightclub) usually presents with signals of a regular frequency. This is not seen in this case. Still, the absence of these features does not rule out external noise because the source of the noise may have been irregular. The most likely explanation is lead or insulation fracture. Although each has unique findings, they can both present with what appears to be randomly sensed noise. Exceptions to this occur when the signals are filtered, as is the case with “noise reversion,” wherein the device detects

80

Case 4  |  Long-QT Syndrome and Multiple ICD Shocks

what it believes to be noise and filters it out. Blanking periods may also lend a regular pattern to irregular noise. In the case of lead fracture, impedance to current flow is increased (since the current flow through the lead conductor), and pacing thresholds may be increased; with insulation failure, impedance is decreased (because the insulation to radial current flow is interrupted). Keep in mind that neither these findings may occur during device interrogation. Frequently, patients have to undergo provocative maneuvering of their arm and shoulder or the pocket itself prior to reproducing either the noise or the abnormal electrical parameters. Thus, it is the stored electrograms from the event as logged by the device that provide the most significant clues to the etiology of the fracture. In this case, a lead fracture had occurred, probably during dancing. Because of the patient’s age, the fractured lead was explanted and a new lead was placed.

81

Case 5  FrequentPalpitations in a Patient with Implantable Cardioverter Defibrillator TEACHING POINTS • Arrhythmias diagnosis algorithm

• “Wobble” phenomenon

Case Presentation A 64-year-old man with a dual-chamber ICD complains of frequent palpitations. Interrogation of the ICD demonstrates multiple episodes of tachycardia defined as ventricular tachycardia (VT) (fibrillation). A representative tracing is shown below (Figure 1).

Figure 1  Top panel: atrial electrogram (A). Middle panel: ventricular electrogram (V). Bottom panel: marker channel (Marker).

83

Case 5  |  Frequent Palpitations in a Patient with ICD

Case Discussion What is the likely rhythm? The tachycardia demonstrates a regular pattern with 1:1 AV relationship. The differential includes SVT with rapid ventricular conduction versus VT with 1:1 VA conduction. The atrial electrogram shows a near-field atrial signal (star) followed by a far-field ventricular signal (arrow). This confirms the lead’s position in the right arial appendage. The appendage drapes over the tricuspid valve. This accounts for the far-field ventricular signals. With regard to the nature of the tachycardia, the most important observation is that of tachycardia “wobble.” The concept of wobble is straightforward: During tachycardia, any alterations in cycle length will be initiated by the tachycardia “driver.” Thus, alterations in A-A and V-V intervals will be first noted in the A-A interval if the tachycardia’s genesis is the atrium (or, at least, not the ventricle). If the wobble is first noted in the V-V intervals, then the origin of the tachycardia is likely the ventricle and diagnosis will be VT (Figure 2).

84

Case 5  |  Frequent Palpitations in a Patient with ICD

figure 2

Turning our attention to the tracing, we can appreciate that A-A intervals of 410 and 390 msec are followed by V-V intervals of the same duration. This suggests supraventricular origin for the tachycardia. The next A-A interval, however, is 330 msec with a subsequent V-V interval

85

Case 5  |  Frequent Palpitations in a Patient with ICD

of 350 msec. Why, if the tachycardia is originating above the ventricle, is the A-A interval of 330 msec not followed by an identical V-V interval? The answer lies in the fact that the AV node has decremental properties. Thus, the V-V interval during SVT may not be able to keep up with the increased atrial rate because of the delay in the AV node. The reverse of this is seen later when a 20 msec increase in the A-A (330 to 350 msec) leads to no change in the V-V interval (330 msec on consecutive V-Vs). The most likely explanation is that the delay in the A-A allows the AV node to recover enough to allow enhanced AV nodal conduction, which compensates for the delay. In other words, the 20 msec delay in the A-A interval has allowed the AV node to conduct 20 msec faster. The next effect is that the V-V interval remains unchanged at 330 msec. This brings us to another important point: wobble is not necessarily reflected by identical changes in intervals. It is the trend in the alterations that is most important. The next image (Figure 2) confirms two observations: (1) the tachycardia is not driven by the ventricle as the atrium is the chamber in which the tachycardia terminates first. This further reinforces the previous conclusion of SVT/AT as the tachycardia mechanism. (2) We can also see that the farfield deflection in the atrial channel is indeed due to ventricular activity.

86

Case 6  Palpitations in a Patient with a Dual-chamber Implantable Cardioverter Defibrillator TEACHING POINTS • Atrial high-rate events • Mode switching

• Pacing timing intervals

Case Presentation A 64-year-old has a dual-chamber ICD (St Jude Current DR VT detection interval 300-400 msec; ventricular fibrillation (VF) detection interval at a cycle length (CL)